For Each Positive Integer N, Let Xn=1/(n+1)+1/(n+2)+⋯+1/(2n). Prove ...
-
- Home
- Questions
- Tags
- Users
- Unanswered
- Teams
Ask questions, find answers and collaborate at work with Stack Overflow for Teams.
Try Teams for free Explore Teams - Teams
-
Ask questions, find answers and collaborate at work with Stack Overflow for Teams. Explore Teams
Teams
Q&A for work
Connect and share knowledge within a single location that is structured and easy to search.
Learn more about Teams For each positive integer $n$, let $x_n=1/(n+1)+1/(n+2)+\cdots+1/(2n)$. Prove that the sequence $(x_n)$ converges. Ask Question Asked 11 years, 1 month ago Modified 1 year, 4 months ago Viewed 27k times 4 $\begingroup$For each positive integer $n$, let $x_n=1/(n+1)+1/(n+2)+\cdots+1/(2n)$. Prove that the sequence $(x_n)$ converges.
This is what I have so far..
Let $\epsilon > 0$ be given to us. We must show that there exists an $N \in \mathbb{N}$ such that $n\ge N \implies \left|\frac{1}{2n} - 1\right| < \epsilon$. Choose $N$ to be any positive integer which is larger than $\frac{1}{\epsilon}$. (So $N>\frac{1}{\epsilon}$.) Then $n\ge N \implies \left|\frac{1}{2n} - 1\right| = |\frac{1-2n}{2n}|$
Share Cite Follow edited Jan 26, 2018 at 17:39 user88319 asked Oct 14, 2013 at 11:28 ArnoldArnold 8396 gold badges16 silver badges30 bronze badges $\endgroup$ 1- $\begingroup$ I need to find a way such to show that |1/(2n) – 1| < ε $\endgroup$ – Arnold Commented Oct 14, 2013 at 11:34
3 Answers
Sorted by: Reset to default Highest score (default) Date modified (newest first) Date created (oldest first) 15 $\begingroup$We write $$x_n=\sum_{k=1}^n\frac{1}{n+k}=\frac{1}{n}\sum_{k=1}^n\frac{1}{1+k/n}\to\int_0^1\frac{dx}{1+x}=\log 2$$
Share Cite Follow answered Oct 14, 2013 at 11:33 user63181user63181 $\endgroup$ 2- 1 $\begingroup$ Big question, why is the integral up to 1? $\endgroup$ – Lilian Hernández Commented Apr 14, 2020 at 5:56
- $\begingroup$ @user63181 after putting x=k/n and dx/dk=1/n i.e dx=dk/n. but in the above integration you used simply dx. How ?? $\endgroup$ – The Iotaa Commented Sep 8, 2021 at 8:45
Hints:
$$\frac12=\frac n{2n}\le\frac1{n+1}+\frac1{n+2}+\ldots+\frac1{2n}\le\frac n{n+1}$$
$$X_{n+1}:=\frac1{n+2}+\frac1{n+3}+\ldots+\frac1{2(n+1)}\ge \frac1{n+1}+\frac1{n+2}+\ldots+\frac1{2n}=:X_n$$
[Edit: Flipping the inequality the right way]
Share Cite Follow edited Jul 28, 2023 at 13:44 Ph C G 4110 bronze badges answered Oct 14, 2013 at 11:35 DonAntonioDonAntonio 214k18 gold badges140 silver badges290 bronze badges $\endgroup$ 4- $\begingroup$ Can we find out to what number does the sequence converge? I know it is a monotonically increasing sequence bounded above by ln(2). Is ln(2) the supremum? Then, the limit would be ln(2). Is it? $\endgroup$ – Swapnil Tripathi Commented Jun 9, 2014 at 19:47
- 2 $\begingroup$ The last inequality is the wrong way around. Note that the number of terms on both sides is not equal, which is clouded by the suggestive notation. $\endgroup$ – WimC Commented Feb 6, 2016 at 18:00
- $\begingroup$ @DonAntonio Nice answer! But you haven't computed the limit. Btw, do you know how to calculate it without using Riemann sums ? $\endgroup$ – Arthur Commented Mar 10, 2023 at 14:32
- $\begingroup$ @Franklin This is very old...and the short answer is no: I don't know how to evaluate the limit without using Riemann sums. To prove the limit exists is done above in hints (bounded monotonic sequence), but the actual value of the limit...I personally can do it only with Riemann integrals. $\endgroup$ – DonAntonio Commented Mar 10, 2023 at 18:14
The sequence is decreasing because $$x_{n+1}-x_n = \frac1{2n+2}+\frac1{2n+1} - \frac1{n+1} = \frac{1}{2(2n+1)(n+1)}\ge 0$$ and obviously, $x_n\le \frac{n}{n+1}\le 1$, so is convergent. For proving that $x_n\to \log 2$ the better idea is bounding $x_n$ by definite integrals of the form $\int_a^b\frac1x\,dx$ (how?).
Share Cite Follow edited Oct 27, 2017 at 6:30 answered Feb 9, 2016 at 13:56 Martín-Blas Pérez PinillaMartín-Blas Pérez Pinilla 42.2k4 gold badges49 silver badges92 bronze badges $\endgroup$ 2- $\begingroup$ $-\frac{1}{n}$ is wrong, it should be $-\frac{1}{n+1}$, shouldn't it? But if so, you end up with $x_{n+1} \gt x_n$, right? Does it converge then? $\endgroup$ – Math for fun Commented Oct 26, 2017 at 21:14
- $\begingroup$ @Flavius, corrected. Thanks. $\endgroup$ – Martín-Blas Pérez Pinilla Commented Oct 27, 2017 at 6:30
You must log in to answer this question.
Not the answer you're looking for? Browse other questions tagged
.- Featured on Meta
- We’re (finally!) going to the cloud!
- More network sites to see advertising test [updated with phase 2]
Linked
2 Calculating the limit of the sum. 0 Evaluate $\lim _{n\to\infty}\left[\frac{1}{n+1}+\frac{1}{n+2}+\cdots \frac{1}{n+n}\right]$ 1 $ \lim \limits_{n \to \infty}(\dfrac{1}{n+1}+\dfrac{1}{n+2}.....\dfrac{1}{n+n})$ 2 How would I find the limit for this equaiton? 3 How do I prove $\frac 34\geq \frac{1}{n+1}+\frac {1}{n+2}+\frac{1}{n+3}+\cdots+\frac{1}{n+n}$ 0 Find upper bound for sequence 0 How can I calculate the result of the limit of this sequence $A_{n}$?Related
4 Prove $x_n$ converges if $x_n$ is a real sequence and $s_n=\frac{x_0+x_1+\cdots+x_n}{n+1}$ converges 5 Let $(x_n)$ be an integer Cauchy sequence. Prove that $x_n$ is eventually constant. 1 Prove that the sequence $(b_n)$ converges 6 Prove that if $\lim \frac{x_{n+1}}{x_n}=L<1$ then it converges to $0$. 1 $\left(x_n\right) \in \mathbb{R}$ which converges to $l$ where $\alpha<x_n$. Show that $\alpha \leq l$. 1 Let $\{x_n\}$ be a sequence in $(0, 1)$ such that $x_n \to 0$. Show that the sequence $\{f(x_n)\}$ converges. 0 Use the definition of convergence to prove that the sequence $\left\{\frac{2n}{3n+2}\right\}$ converges. 7 T/F: If $(x_n)$ is a positive real sequence s.t. $\sum x_n$ converges, then $\exists N$ s.t. $x_{\left\lceil\frac{1}{x_N }\right\rceil}<\frac{1}{N}.$Hot Network Questions
- Are prenups legally binding in England?
- Most Efficient Glide: Pitch Up or Level Flight to Bleed Airspeed
- Line breaks do not fit well in mathmode
- How to inflict self damage anywhere in Fallout 2?
- Why should C++ uint8_t data not be printable?
- How to demystify why my degree took so long on my CV
- A fantasy movie with two races, "Big Ones" (=us) and smaller ones, about saving a newborn baby from a cruel queen
- A Title "That in Aleppo Was"
- Is ATL-98 Carvair still alive in the US?
- What could be the potential risk of installing this "ATB" handlebar on a road bike?
- The sum of multiple irrational numbers can be rational, even when they're not conjugates. Is this normal?
- Looking for short story about detectives investigating a murder in the future
- Formative alternative to midterms for a large class
- Convergence of random functions
- Can you omit から directly connecting plain forms?
- What does "inside" refer to when we say that the electric field inside a conductor is 0?
- cartridge style bottom bracket temperature range
- How do you write a page-centered abstract in a two column document
- Translating Russian "не то, не то" into English
- Embossing a model's texture onto its mesh
- Connectedness of intersection of two sets
- range has lost one leg of 220
- What is the name of the lady with the white blouse?
- Why do I have to reboot to activate a kernel upgrade even though live kernel updates are enabled?
To subscribe to this RSS feed, copy and paste this URL into your RSS reader.
Từ khóa » Xn=1+2n+n^2/n^2
-
[PDF] Sequences And Series
-
[PDF] The Limit Of A Sequence - MIT Mathematics
-
[PDF] Tests For Convergence Of Series 1) Use The Comparison Test To ...
-
[PDF] Calculus II, Section 11.8, #24 Power Series Find The Radius Of ...
-
Sequences - Finding A Rule - Math Is Fun
-
[PDF] Section 12.9, Problem 38 - Math
-
[PDF] Proof By Induction - University Of Plymouth
-
Is This Sequence Convergent X_n=1/ (n+1) +1/(n+2) +…+1/(2n)?
-
Solutions To Exercises
-
[PDF] MATHEMATICAL INDUCTION SEQUENCES And SERIES
-
[PDF] 2n 3n + N3 . Answer
-
Infinite Series Convergence And Divergence Example With SUM((2n ...
-
[PDF] Exam-3 Solutions, Math 10560 1. Find The Sum Of The Following Series
-
[PDF] SOLUTIONS TO EXAM 3 Problem 1. Find The Sum Of The Following ...